LSAT and Law School Admissions Forum

Get expert LSAT preparation and law school admissions advice from PowerScore Test Preparation.

User avatar
 Dave Killoran
PowerScore Staff
  • PowerScore Staff
  • Posts: 5852
  • Joined: Mar 25, 2011
|
#41309
Complete Question Explanation
(The complete setup for this game can be found here: lsat/viewtopic.php?t=15710)

The correct answer choice is (C)

This is an unusual partial List question.

Answer choice (A) is incorrect because when M is added third then L must be added sixth.

Answer choice (B) is incorrect because the proposed scenario would force K or T to be added fifth, and that would be a violation of the third rule.

Answer choice (C) is the correct answer.

Answer choice (D) is incorrect because if Z is added first then L cannot be added last.

Answer choice (E) is incorrect because if Z is added first then M cannot be added second.

Note that answer choices (B), (D), and (E) are all violations of the inferences discussed above.
 Katya W
  • Posts: 42
  • Joined: Dec 03, 2019
|
#74395
Hi, could we please see how one would attack this problem? Do you make skeleton diagrams for each answer choice that is not an obvious elimination? I got this question correct, but I’m a visual learner so when there are no explanations of how a problem is tackled to find the correct answer that sets me back. Thank you!
 Jeremy Press
PowerScore Staff
  • PowerScore Staff
  • Posts: 1000
  • Joined: Jun 12, 2017
|
#74459
Hi Katya,

You've basically got the right procedure in mind here! This is what we call a "partial list" question, because each answer choice contains a partial list of the foods in the order they're added (leaving the other foods uncertain). Approach it the way you would a "Global List" question, the question that ordinarily begins each game, where the answer choices list complete potential solutions to the game. Use each rule, in order, and look for answer choices that would directly violate that rule.

The first rule is directly violated by answer choice A, because, although mushrooms are added third, lentils are not added last. The second rule is directly violated by answer choice D, because, although zucchini are added first, lentils could not be added before onions (since the answer choice states that lentils are added last). The last rule is directly violated by answer choice E, because answer choice E would require the mushrooms to be added before both the tomatoes and the kale (rather than just one of them, as the last rule requires).

That process of looking for direct rule violations leaves you with answer choices B and C. Test one of them. In other words, try to fill out a complete solution consistent with the rules. If you're able to create a complete solution for that answer, it is the correct answer. If you're not able to create a complete solution for that answer, it's incorrect and the other answer is correct. When I'm in this position, I usually just test the first available answer, in this case answer choice B. As soon I try to solve that answer, I can see the problem. Mushrooms have to be added before either tomatoes or kale. So in answer choice B, tomatoes or kale would have to be added fifth. But that breaks the third rule. Thus, answer choice C is correct by process of elimination.

I hope this helps!

Jeremy
 Katya W
  • Posts: 42
  • Joined: Dec 03, 2019
|
#74635
Thank you Jeremy! Your responses are very easy to understand!
 leslie7
  • Posts: 73
  • Joined: Oct 06, 2020
|
#80743
Which one of the following could be an accurate partial ordering of the foods added to the broth?
A. Lentils second, mushrooms third

I did end up getting this question correct and I think I eliminated A for a reason other than what was explained in the textbook and I would like some clarity on that explanation.

Explanation:
A is incorrect because when M is added third then L must be added sixth

In other words If M3-->L6. Which is correctly a conditional in the rules.

My question is the following:

If M3--> L6 = /L6--->/M3 however, based on my understanding of sufficiency/necessity - it could be the case that If L6 --> M3 . So if the list says Lentils second and mushrooms third (L2--> M3 - technically that could be permissible correct?) So, how do you know in that list in the answer choices which one is the necessary vs sufficient condition?

In this case I assumed that since Lentils were being placed second "first" on the list than it was acceptable for Mushrooms to be placed third (accepting that lentils 2nd was the sufficient condition)

Some/ any clarity would be greatly appreciated. I hope my question was clear.
 Jeremy Press
PowerScore Staff
  • PowerScore Staff
  • Posts: 1000
  • Joined: Jun 12, 2017
|
#80757
Hi leslie,

The application of a conditional rule doesn't depend on what order the variables appear in on the line. Even though L is before M in the ordering in answer choice A, that doesn't mean the rule doesn't apply. Rather, a conditional rule applies (it "triggers," as I sometimes say) whenever the sufficient condition is part of the diagram. So, for answer choice A, M was added third. That means the rule applies (the sufficient condition "triggers," because it's part of the diagram). And that means the necessary condition must happen as well. So, that means L must be added sixth as well. L can't be added second, because the conditional rule forces them to be added sixth. So answer choice A is not a possible partial ordering.

Now, it's true that if L were sixth, you could potentially have M in 3, or you could have M somewhere else other than 3. But that's not the circumstance here. Here, L is in 2. That means L is not 6th, and so from that perspective the contrapositive would also mean that M could not be third. Either way you look at it, the rule is violated in answer choice A!

I hope this helps!
 leslie7
  • Posts: 73
  • Joined: Oct 06, 2020
|
#83135
Jeremy Press wrote: Wed Nov 04, 2020 4:16 pm Hi leslie,

The application of a conditional rule doesn't depend on what order the variables appear in on the line. Even though L is before M in the ordering in answer choice A, that doesn't mean the rule doesn't apply. Rather, a conditional rule applies (it "triggers," as I sometimes say) whenever the sufficient condition is part of the diagram. So, for answer choice A, M was added third. That means the rule applies (the sufficient condition "triggers," because it's part of the diagram). And that means the necessary condition must happen as well. So, that means L must be added sixth as well. L can't be added second, because the conditional rule forces them to be added sixth. So answer choice A is not a possible partial ordering.

Now, it's true that if L were sixth, you could potentially have M in 3, or you could have M somewhere else other than 3. But that's not the circumstance here. Here, L is in 2. That means L is not 6th, and so from that perspective the contrapositive would also mean that M could not be third. Either way you look at it, the rule is violated in answer choice A!

I hope this helps!
Hi Jeremy i appreciate this response but would you be able to clarify whether this is still a possible outcome L2->M3 ?
( I think it still is)

The reason I eliminated A was because if L2 and M3 and T/K must come before M then 1 MUST be either T/K but since that means Z is not in 1 than O must come before L meaning it should be in slot 1 but since slot 1 must be occupied by T/K then we come into a roadblock. Therefore L cannot be in 2 and M cannot be in three in terms of a list of partial ordering.
 Adam Tyson
PowerScore Staff
  • PowerScore Staff
  • Posts: 5153
  • Joined: Apr 14, 2011
|
#84123
It looks like you are having two problems with conditional reasoning here, leslie7:

When M is 3rd, L must be 6th, so if L is anywhere other than 6th (which includes L being 2nd), M absolutely cannot be 3rd. That is the contrapositive. So there is no way L could be 2nd and have M be 3rd. That would be a direct violation of the rule about what absolutely must happen any time M is 3rd.

And then this:
that means Z is not in 1 than O must come before L
This is a Mistaken Negation of the second rule. If Z is 1, L must be before O, but that does not mean that if Z is NOT 1 then O is before L. The Necessary Condition of L before O can happen whether the Sufficient Condition of Z first happens or not. Necessary Conditions can always happen independent of the Sufficient Condition. It's only when the Sufficient Condition occurs that the Necessary Condition has no choice and must occur.

Keep practicing those conditionals until they come automatically!

Get the most out of your LSAT Prep Plus subscription.

Analyze and track your performance with our Testing and Analytics Package.